428 views
1 votes
1 votes
Ques: Can Round Robin algorithm be non-preemptive ?

My answer: RR with largest Burst Time as Time Quantum is FCFS in nature and its non-preemptive

Am I right ? Please corret me if am wrong.

1 Answer

1 votes
1 votes
If tq= q, where q is very large, but at the same time, first process happens to be of burst =q+1, then just for 1 unit time it would get preempted making the algorithm preemptive.
reshown by